Diễn Đàn MathScopeDiễn Đàn MathScope
  Diễn Đàn MathScope
Ghi Danh Hỏi/Ðáp Community Lịch

Go Back   Diễn Đàn MathScope > Sơ Cấp > Tài Liệu > Đề Thi > Đề Thi HSG Cấp Tỉnh ở Việt Nam

News & Announcements

Ngoài một số quy định đã được nêu trong phần Quy định của Ghi Danh , mọi người tranh thủ bỏ ra 5 phút để đọc thêm một số Quy định sau để khỏi bị treo nick ở MathScope nhé !

* Nội quy MathScope.Org

* Một số quy định chung !

* Quy định về việc viết bài trong diễn đàn MathScope

* Nếu bạn muốn gia nhập đội ngũ BQT thì vui lòng tham gia tại đây

* Những câu hỏi thường gặp

* Về việc viết bài trong Box Đại học và Sau đại học


Trả lời Gởi Ðề Tài Mới
 
Ðiều Chỉnh Xếp Bài
Old 01-10-2014, 09:00 AM   #1
namdung
Administrator

 
Tham gia ngày: Feb 2009
Đến từ: Tp Hồ Chí Minh
Bài gởi: 1,343
Thanks: 209
Thanked 4,066 Times in 778 Posts
Gửi tin nhắn qua Yahoo chát tới namdung
Đề thi các trường và các tỉnh năm học 2014-2015 - Lời giải và bình luận

Hôm nay là ngày 1/10, ngày tốt để bắt đầu một công việc mà chúng ta đã làm đều đặn nhiều năm qua: Giải và bình luận đề thi các trường và các tỉnh.

Cuối tháng 9 vừa qua đã có nhiều trường và nhiều tỉnh thi chọn đội tuyển như Đà Nẵng, Bắc Giang, Quảng Trị, Ninh Bình, Hà Tĩnh, Đà Lạt, Đồng Nai, Sóc Trăng, PTNK ... và sắp tới hàng loạt các trường, các tỉnh sẽ thi. Năm nay chúng ta sẽ đổi mới một chút trong việc giải và bình luận. Ta sẽ coi trọng hơn về chất lượng chứ không chạy đua theo số lượng. Vì thế, ta sẽ chọn lọc những bài hay, có ý nghĩa học thuật nhiều hơn để bình luận và thật chi tiết, với cả những vấn đề xung quang, những sai lầm thường gặp. Như vậy các bạn học sinh và các thầy cô giáo sẽ thu nhận được nhiều hơn là đơn thuần một lời giải.

Tôi xin chọn lọc trước ở đây một số bài để các bạn vào giải và bình luận. Các bạn cũng có thể đề xuất các bài toán khác cần có bình luận. Để chủ đề không bị loãng, mong các bạn chọn lọc kỹ một chút. Chú ý là ta không quan tâm nhiều đến số lượng. Quan điểm của chúng ta là không chỉ chọn những bài khó và mới, mà là những bài qua đó có thể giảng giải cho các bạn học sinh được nhiều nhất.

1. (Đồng Tháp) Cho x, y, z là các số thực thoả mãn điều kiện $x + y + z = 5, x^2 + y^2 + z^2 = 9$ . Tìm giá trị lớn nhất và giá trị nhỏ nhất của biểu thức $ P = x^3+ y^3 + z^3 $.

2. (Đồng Tháp) Có một số số thực dương, mỗi số không lớn hơn 0,5 và tổng của chúng không vượt quá 2,5.
a) Chứng minh rằng ta có thể chia các số thành 4 nhóm, mỗi nhóm có tổng các số không vượt quá 1.
b) Khẳng định trên còn đúng không nếu thay 4 nhóm bằng 3 nhóm?

3. (Quảng Trị) Chứng minh bất đẳng thức sau $3(x^2-x+1)(y^2-y+1) \ge 2(x^2y^2-xy+1)$ với mọi x, y thực.

4. (Quảng Trị) Ban đầu trên bảng điện tử hiển thị hai số phân biệt a, b. Sau mỗi giây, bảng sẽ tự động hiển thị thêm các số n nếu n chưa có trên bảng và là tổng của hai số đã có trên bảng. Hãy xét xem số 2014 có được hiển thị trên bảng không, nếu có thì sau thời gian ít nhất bao lâu (kể từ thời điểm ban đầu), trong các trường hợp sau a) a = 3, b = 12; b) a = 1, b = 2.

5. (PTNK) Cho a, b, c là các số thực dương thỏa mãn điều kiện $ (a+1)(b+1)(c+1) = 1 + 4abc $ Chứng minh rằng ta có bất đẳng thức $ a + b + c \le 1 + abc $

6. (PTNK) Tìm tất cả các hàm số f: N* --> N* thỏa mãn hệ thức $ f(f(n)/n) = n^2 $ với mọi n nguyên dương. N* ký hiệu tập hợp các số nguyên dương.

7. (Sóc Trăng) Sau khi thi tốt nghiệp trung hoc phổ thông, các em học sinh khối 12 của trường trung học phổ thông MTT hẹn nhau mỗi năm họp mặt 1 lần vào ngày hội trại của trường. Tuy nhiên các thành viên không có mặt đủ theo giao ước ban đầu mà chỉ có đúng 20 thành viên tham dự mỗi lần họp mặt. Qua 10 lần họp mặt không có 2 thành viên nào dự họp mặt với nhau quá một lần. Hỏi khối 12 năm học đó của trường trung học phổ thông MTT có ít nhất là bao nhiêu học sinh?

8. (Đà Nẵng) Tại một hội nghị quốc tế, các đại biểu tham dự biết ít nhất một trong 3 thứ tiếng Anh, Pháp, Đức. Biết rằng có đúng 50 đại biểu biết tiếng Anh, đúng 50 đại biểu biết tiếng Pháp và đúng 50 đại biểu biết tiếng Đức. Chứng minh rằng có thể chia các đại biểu thành 5 nhóm sao cho trong mỗi nhóm có đúng 10 đại biểu biết tiếng Anh, đúng 10 đại biểu biết tiếng Pháp và đúng 10 đại biểu biết tiếng Đức.

9. (Đà Nẵng) Các ô vuông của bảng vuông kích thước 10 x 10 được tô bằng các màu trắng và đen sao cho trên mỗi hàng cũng như trên mỗi cột đều có đúng ba ô được tô màu đen. Chứng minh rằng trong mọi cách tô như vậy luôn có thể chọn ra 10 ô được tô màu đen sao cho không có hai ô nào được chọn cùng hàng hoặc cùng cột.

10. (Đội tuyển Lương Thế Vinh, Đồng Nai) Tìm tất cả các hàm số f: R --> R thỏa mãn điều kiện f(f(x) - y) + f(x+y) = 2x với mọi x, y thuộc R.

11. (???) Cho dãy số $ u_n $ được xác định bởi $ u_1 = \frac{21}{10} $, $ u_{n+1} = \frac{u_n-2+\sqrt{u_n^2+8u_n-4}}{2}$ với mọi n = 1, 2, 3, ... Đặt $v_n = \frac{1}{u_2^2-4} + ... + \frac{1}{u_{n+1}^2-4} $ Tìm $ lim v_n $ khi n dần đến vô cùng.

12. (Đà Lạt) Cho a, b, c là các số thực dương thỏa mãn điều kiện a + b + c = 3. Chứng minh rằng ta có bất đẳng thức $a^3 + b^3 + c^3 + 2(\frac{1}{a} + \frac{1}{b} + \frac{1}{c}) \ge 3(ab+bc+ca)$

13. (Đà Lạt) Từ các chữ số 1, 3, 5, 9 có thể lập được bao nhiêu số tự nhiên chia hết cho 3, mỗi số có 2014 chữ số.

Rất mong sự hưởng ứng của các thầy cô, các bạn sinh viên, các cựu IMO, cựu VMO và các bạn học sinh.
[RIGHT][I][B]Nguồn: MathScope.ORG[/B][/I][/RIGHT]
 

thay đổi nội dung bởi: namdung, 01-10-2014 lúc 03:35 PM Lý do: Latex
namdung is offline   Trả Lời Với Trích Dẫn
The Following 27 Users Say Thank You to namdung For This Useful Post:
1110004 (01-10-2014), 9nho10mong (02-10-2014), baotram (01-10-2014), chelseaMS (01-10-2014), chuyentoanltt (03-10-2014), davidsilva98 (01-10-2014), DenisO (01-10-2014), doanthanh (05-10-2014), einstein1996 (01-10-2014), falcaono1 (02-10-2014), HoangHungChels (01-10-2014), huy230499 (17-01-2015), huynhcongbang (07-10-2014), Juliel (02-10-2014), lethuc_92 (17-12-2014), lupanh7 (08-11-2014), nhatduyt1k24 (27-10-2014), phamvanhuy (01-10-2014), phucbentre (15-10-2014), Saruka 01 (01-10-2014), thaygiaocht (01-10-2014), Thmcuongvn (05-10-2014), Trung_Nhu0602 (26-10-2014), tson1997 (01-10-2014), vantienducdh (26-10-2014), whatever2507 (05-10-2014), ĐồngLê 97 (04-10-2014)
Old 01-10-2014, 04:28 PM   #2
tson1997
+Thành Viên+
 
tson1997's Avatar
 
Tham gia ngày: Jun 2012
Đến từ: K46 T1 chuyên SP
Bài gởi: 46
Thanks: 42
Thanked 51 Times in 24 Posts
Bài 2 (Đồng Tháp)

a/ Xét phương trình $a_1+a_2+..+a_d=2,5$ (d nguyên dương >4 bất kì, $0< a_i <2,5$ )
Ta sẽ cminh bằng quy nạp theo d rằng có thể chia được d số trên thành 4 nhóm sao cho tổng mỗi nhóm không vượt quá 1
Với $d \leq 8$ thì ta chia d số làm 4 nhóm,mỗi nhóm không quá 2 số (t/m)
Với $d = 9$ ,ta có:
$(a_1+a_2+a_3)+(a_4+a_5+a_6)+(a_7+a_8+a_9) = 2,5$
Như vậy,tồn tại 1 trong 3 tổng trên < 1
Giả sử $a_1+a_2+a_3 < 1$
Như vậy , 9 số $a_1;a_2;..;a_9$ có thể chia được làm 4 nhóm:
{$a_1;a_2;a_3$};{$a_4;a_5$};{$a_6;a_7$};{$a_8;a_9$ }
tổng của mỗi nhóm đều k vượt quá 1

$d \geq 10$: Ta có:
$(a_1+a_2)+(a_3+a_4)+...+(a_9+a_10) \leq 2,5$
Như vậy,luôn có 1 trong 5 tổng trên có giá trị k vượt quá 0,5
giả sử $a_1+a_2 \leq 0,5$
Ta "gộp" $a_1,a_2$ thành $a_1+a_2$
Theo giả thiết quy nạp ta có thể chia được bộ d-1 số $a_1+a_2;a_3;..;a_d$ thành 4 nhóm,như vậy ta cũng chia được bộ d số $a_1;a_2;..;a_d$ thành 4 nhóm

Vậy ta có đpcm

b/ Xét $a_1=a_2=..=a_7=\frac{5}{14}$
Do chỉ có 3 nhóm nên theo định lý Dirichlet luôn có 1 nhóm có không ít hơn 3 phần tử,tổng của nhóm đó là $3.\frac{5}{14}=\frac{15}{14} > 1$
Như vậy khẳng định không đúng nếu thay 4 bởi 3




[RIGHT][I][B]Nguồn: MathScope.ORG[/B][/I][/RIGHT]
 
tson1997 is offline   Trả Lời Với Trích Dẫn
The Following 5 Users Say Thank You to tson1997 For This Useful Post:
namdung (02-10-2014), Raul Chavez (02-10-2014), thaygiaocht (01-10-2014), Thmcuongvn (05-10-2014), whatever2507 (05-10-2014)
Old 01-10-2014, 05:23 PM   #3
einstein1996
Senior Member
 
Tham gia ngày: Nov 2011
Đến từ: việt nam
Bài gởi: 103
Thanks: 77
Thanked 43 Times in 28 Posts
Bài của Đồng Tháp từng đăng trên MS một lần rồi [Only registered and activated users can see links. ]
[RIGHT][I][B]Nguồn: MathScope.ORG[/B][/I][/RIGHT]
 
einstein1996 is offline   Trả Lời Với Trích Dẫn
The Following 2 Users Say Thank You to einstein1996 For This Useful Post:
thinhrost1 (30-08-2016), tson1997 (01-10-2014)
Old 01-10-2014, 06:28 PM   #4
nguyentatthu
Super Moderator
 
Tham gia ngày: Nov 2007
Đến từ: BH
Bài gởi: 212
Thanks: 135
Thanked 345 Times in 92 Posts
File tổ hợp
[RIGHT][I][B]Nguồn: MathScope.ORG[/B][/I][/RIGHT]
 
File Kèm Theo
Kiểu File : doc VMO 2015 To hop.doc (56.0 KB, 458 lần tải)
nguyentatthu is offline   Trả Lời Với Trích Dẫn
The Following 6 Users Say Thank You to nguyentatthu For This Useful Post:
einstein1996 (01-10-2014), huynhcongbang (07-10-2014), materazzi (02-10-2014), namdung (02-10-2014), thaygiaocht (02-10-2014), tson1997 (02-10-2014)
Old 02-10-2014, 12:39 AM   #5
doxuantung97
+Thành Viên+
 
Tham gia ngày: Aug 2012
Đến từ: Toán 1 K46 Chuyên Sư Phạm
Bài gởi: 49
Thanks: 19
Thanked 24 Times in 12 Posts
Bài 11:
Giả thiết ta có $(2u_{n+1}-u_{n}+2)^{2}= u_{n}^{2}+8u_{n}-4$
$\Rightarrow u_{n+1}^{2}= u_{n}u_{n+1}+3u_{n}-2u_{n+1}-2$ (1)
$\Rightarrow u_{n+1}^{2}-4= (u_{n}-2)(u_{n+1}+3)$ (2)
Mà từ (1) lại có $(u_{n+1}+3)(u_{n+1}-u_{n})= u_{n+1}-2$
$\Rightarrow u_{n+1}+3= \frac{u_{n+1}-2}{u_{n+1}-u_{n}}$
Thay vào (2) có $u_{n+1}^{2}-4= \frac{(u_{n+1}-2)(u_{n}-2)}{u_{n+1}-u_{n}}$
$\Rightarrow \frac{1}{u_{n+1}^{2}-4}= \frac{1}{u_{n}-2}-\frac{1}{u_{n+1}-2}$
$\Rightarrow v_{n}= \frac{1}{u_{1}-2}-\frac{1}{u_{n+1}-2}$
Dễ có $u_{n}\rightarrow +\infty $ nên tính được lim$v_{n}$
[RIGHT][I][B]Nguồn: MathScope.ORG[/B][/I][/RIGHT]
 
doxuantung97 is offline   Trả Lời Với Trích Dẫn
The Following 5 Users Say Thank You to doxuantung97 For This Useful Post:
falcaono1 (19-10-2014), namdung (02-10-2014), Thmcuongvn (05-10-2014), truongson2007 (03-10-2014), tson1997 (02-10-2014)
Old 02-10-2014, 10:46 AM   #6
nguyentatthu
Super Moderator
 
Tham gia ngày: Nov 2007
Đến từ: BH
Bài gởi: 212
Thanks: 135
Thanked 345 Times in 92 Posts
Bài 5.
Đặt $x=\frac{1}{a},y=\frac{1}{b},z=\frac{1}{c}$ ta có $x+y+z+xy+yz+zx=3$
Ta cần chứng minh $$xy+yz+zx\le xyz+1\Leftrightarrow x+y+z+xyz\ge 2$$ (*).
Nếu $x+y+z\ge 2$ thì (*) đúng.
Xét $x+y+z<2$. Đặt $p=x+y+z,q=xy+yz+zx,r=xyz$.
Ta có $$r\ge \frac{p\left( 4q-{{p}^{2}} \right)}{9}=\frac{p\left( 12-4p-{{p}^{2}} \right)}{9}$$

Do đó $$p+r\ge p+\frac{p\left( 12-4p-{{p}^{2}} \right)}{9}$$ .
Ta chứng minh $$p+\frac{p\left( 12-4p-{{p}^{2}} \right)}{9}\ge 2\Leftrightarrow {{p}^{3}}+4{{p}^{2}}-21p+18\le 0$$
$$\Leftrightarrow \left( p-2 \right)\left( {{p}^{2}}+6p-9 \right)\le 0 (1)$$ .
Ta có $p-2<0$ và ${{p}^{2}}\ge 3q=3\left( 3-p \right)\Rightarrow {{p}^{2}}+3p\ge 9\Rightarrow {{p}^{2}}+6p-9>0$.
Từ đó ta suy ra (1) đúng.

[RIGHT][I][B]Nguồn: MathScope.ORG[/B][/I][/RIGHT]
 
nguyentatthu is offline   Trả Lời Với Trích Dẫn
The Following 9 Users Say Thank You to nguyentatthu For This Useful Post:
davidsilva98 (05-10-2014), DenisO (02-10-2014), falcaono1 (02-10-2014), Juliel (02-10-2014), Livetolove2207 (10-12-2014), namdung (02-10-2014), Raul Chavez (02-10-2014), son235 (02-10-2014), thaygiaocht (02-10-2014)
Old 02-10-2014, 11:08 AM   #7
9nho10mong
+Thành Viên+
 
Tham gia ngày: Dec 2013
Bài gởi: 13
Thanks: 9
Thanked 38 Times in 7 Posts
Trích:
Nguyên văn bởi namdung View Post
5. (PTNK) Cho a, b, c là các số thực dương thỏa mãn điều kiện $ (a+1)(b+1)(c+1) = 1 + 4abc $ Chứng minh rằng ta có bất đẳng thức $ a + b + c \le 1 + abc $
Đặt $ \displaystyle x=a+ b >0 \ ; \ y = ab > 0 \ ; \ x^2 \ge 4y $ , từ điều kiện đề bài có
$$ cx + x + y + c = 3cy $$
Hay
$$ c \left( 3y - x - 1 \right) = x+y $$
Từ $ \displaystyle c > 0 , x+y > 0 $ , suy ra
$$ 3y > x+1 $$

$$ c = \frac{x+y}{3y-x-1} $$
Từ $ \displaystyle x^2 \ge 4y $ có
$$ \frac{3x^2}{4} \ge 3y > x+1 $$
Suy ra
$$ x > 2 $$
Ta thấy
$$ abc+1 - \left( a+b+c \right) = \frac{ \left( y-x+1 \right)^2 + x-2}{3y-x-1} > 0 $$
Từ đó có điều phải chứng minh .

Trích:
Nguyên văn bởi namdung View Post
3. (Quảng Trị) Chứng minh bất đẳng thức sau $3(x^2-x+1)(y^2-y+1) \ge 2(x^2y^2-xy+1)$ với mọi x, y thực.

$$ P = 3 \left( x^2-x+1 \right) \left( y^2-y+1 \right) - 2 \left( x^2y^2 - xy+1 \right) \\
= \left( y^2-3y+3 \right) x^2 + \left( 5y-3y^2-3 \right) x + 3y^2-3y+1 $$
Có $ \displaystyle y^2 -3y+3 > 0 $ và
$$ \Delta = -3 \left( y^2- 3y+1 \right)^2 \le 0 $$
Nên
$$ P \ge 0 $$
Đó là điều cần chứng minh .
[RIGHT][I][B]Nguồn: MathScope.ORG[/B][/I][/RIGHT]
 

thay đổi nội dung bởi: 9nho10mong, 02-10-2014 lúc 11:17 AM
9nho10mong is offline   Trả Lời Với Trích Dẫn
The Following 11 Users Say Thank You to 9nho10mong For This Useful Post:
falcaono1 (02-10-2014), hoangthuygiang (04-09-2016), huy230499 (17-01-2015), Livetolove2207 (10-12-2014), namdung (02-10-2014), son235 (02-10-2014), thaygiaocht (02-10-2014), thinhrost1 (30-08-2016), Thmcuongvn (05-10-2014), tson1997 (02-10-2014), tuankiet.tvk (05-10-2014)
Old 02-10-2014, 05:02 PM   #8
tson1997
+Thành Viên+
 
tson1997's Avatar
 
Tham gia ngày: Jun 2012
Đến từ: K46 T1 chuyên SP
Bài gởi: 46
Thanks: 42
Thanked 51 Times in 24 Posts
Trích:
Nguyên văn bởi namdung View Post

1. (Đồng Tháp) Cho x, y, z là các số thực thoả mãn điều kiện $x + y + z = 5, x^2 + y^2 + z^2 = 9$ . Tìm giá trị lớn nhất và giá trị nhỏ nhất của biểu thức $ P = x^3+ y^3 + z^3 $.
Ta có giả thiết tương đương với : $x+y+z=5$ và $xy+yz+zx=8$
Như vậy,x,y,z là 3 nghiệm của $f(t)=t^3-5t^2+8t-xyz$
Suy ra :
$x^3+y^3+z^3=5(x^2+y^2+z^2)-8(x+y+z)+3xyz=5+3xyz$
--> ta sẽ chuyển bài toán về tim min và max của $xyz$:
Ta có : $f'(t)=3t^2-10t+8$
$f'(t)=0 \Leftrightarrow$ $t=2$ v $t=\frac{4}{3}$
Lập bảng biến thiên,ta có $f(t)$ có 3 nghiệm thực $x;y;z$ khi và chỉ khi $f(\frac{4}{3}) \geq 0$ và $f(2) \leq 0$
Từ đây suy ra $\frac{112}{27} \geq xyz \geq 4$

Suy ra $\frac{157}{9} \geq x^3+y^3+z^3 \geq 17$

Dấu "=" xảy ra :
Min : $x=1;y=2;z=2$
Max: $x=\frac{7}{3} ;y=z=\frac{4}{3}$







Bài 12 : Ta chứng minh $VP \geq 9 \geq VT$:
Ta có : $a^3+2 \geq 3a$ ;$b^3+2 \geq 3b$ ;$c^3+2 \geq 3c$
$\frac{1}{a}+\frac{1}{b}+\frac{1}{c} \geq \frac{9}{a+b+c}=3$
Suy ra $VP = a^3+b^3+c^3+2.(\frac{1}{a}+\frac{1}{b}+\frac{1}{c} ) \geq 3a-2+3b-2+3c-2+2.3 = 9$
$VT= 3(ab+bc+ca) \leq (a+b+c)^2 = 9$
Vậy ta có đpcm
[RIGHT][I][B]Nguồn: MathScope.ORG[/B][/I][/RIGHT]
 

thay đổi nội dung bởi: tson1997, 02-10-2014 lúc 05:25 PM
tson1997 is offline   Trả Lời Với Trích Dẫn
The Following 6 Users Say Thank You to tson1997 For This Useful Post:
DenisO (04-01-2015), falcaono1 (02-10-2014), huy230499 (17-01-2015), Livetolove2207 (10-12-2014), namdung (04-10-2014), osp (04-10-2014)
Old 02-10-2014, 05:28 PM   #9
9nho10mong
+Thành Viên+
 
Tham gia ngày: Dec 2013
Bài gởi: 13
Thanks: 9
Thanked 38 Times in 7 Posts
Trích:
Nguyên văn bởi namdung View Post
1. (Đồng Tháp) Cho x, y, z là các số thực thoả mãn điều kiện $x + y + z = 5, x^2 + y^2 + z^2 = 9$ . Tìm giá trị lớn nhất và giá trị nhỏ nhất của biểu thức $ P = x^3+ y^3 + z^3 $.
Từ điều kiện đề bài có
$$ x+y = 5-z \ ; \ x^2+y^2 = 9-z^2 \ge 0 $$
Suy ra
$$ xy = \frac{ \left( x+y \right)^2 - x^2-y^2}{2} = z^2 - 5 z + 8$$
Từ $ \displaystyle 9 \ge z^2 $ và bất đẳng thức quen thuộc $ \displaystyle \left( x + y \right)^2 \ge 4xy $ có
$$ 1 \le z \le \frac{7}{3} $$
Ta thấy
$$ P = z^3 + \left( x + y \right)^3 - 3xy \left( x+y \right) = 3z^3-15z^2 + 24 z + 5 $$
Với $ \displaystyle 1 \le z \le \frac{7}{3} $ có
$$ \frac{157}{9} - P = \frac{\left( 7-3z \right) \left( 3z-4 \right)^2}{9} \ge 0 $$
$$ P - 17 = 3 \left( z -1 \right) \left( z-2 \right)^2 \ge 0 $$

$$ \frac{157}{9} \ge P \ge 17 $$
Tại $ \displaystyle x=y= 2 \ ; \ z =1 $ thì $ \displaystyle P =17 $ .

Tại $ \displaystyle x=y= \frac{4}{3} \ ; \ z = \frac{7}{3} $ thì $ \displaystyle P = \frac{157}{9} $ .

Vậy
$$ \max P =\frac{157}{9} \ ; \ \min P = 17 $$
[RIGHT][I][B]Nguồn: MathScope.ORG[/B][/I][/RIGHT]
 
9nho10mong is offline   Trả Lời Với Trích Dẫn
The Following 3 Users Say Thank You to 9nho10mong For This Useful Post:
huy230499 (17-01-2015), namdung (04-10-2014), osp (04-10-2014)
Old 02-10-2014, 07:27 PM   #10
Short_list
+Thành Viên+
 
Tham gia ngày: May 2012
Đến từ: Tp.HCM
Bài gởi: 85
Thanks: 12
Thanked 79 Times in 32 Posts
Trích:
Nguyên văn bởi namdung View Post
1. (Đồng Tháp) Cho x, y, z là các số thực thoả mãn điều kiện $x + y + z = 5, x^2 + y^2 + z^2 = 9$ . Tìm giá trị lớn nhất và giá trị nhỏ nhất của biểu thức $ P = x^3+ y^3 + z^3 .$
Đặt $x=a+\frac{5}{3},\,y=b+\frac{5}{3},\,z=c+\frac{5}{ 3},$ khi đó $a+b+c=0,\, a^2+b^2+c^2=\frac{2}{3},$ và
\[P=a^3+b^3+c^3+\frac{155}{9}=3abc+\frac{155}{9}.\]
Bài toán quy về tìm GTLN và GTNN của $F=abc$ với $a+b+c=0$ và $a^2+b^2+c^2=\frac{2}{3}.$

Ta chứng minh bài toán tổng quát sau đây:

Trích:
Với mọi số thực $a,\, b,\, c$ thỏa mãn $a+b+c=3p$ và $a^2+b^2+c^2=3p^2+6t^2,\;(t\ge 0)$ thì
$$(p+t)^2(p-2t)\le abc \le (p-t)^2(p+2t)$$
Thật vậy, áp dụng bất đẳng thức Cauchy-Schwarz, ta có
\[3p^2+6t^2=a^2+b^2+c^2 \ge a^2+\frac{(b+c)^2}{2}=a^2+\frac{(3-a)^2}{2}.\]
Suy ra $p-2t\le a \le p+2t.$ Tương ta được $p-2t\le a,\, b,\, c \le p+2t.$ Suy ra tiếp
\[(a-p-2t)(b-p-2t)(c-p-2t) \le 0, \quad (1)\]
\[(a-p+2t)(b-p+2t)(a-p+2t) \ge 0. \quad (2)\]
Khai triển (1) ta được $abc \le (p-t)^2(p+2t).$ Khi triển (2) ta được $(p+t)^2(p-2t) \le abc.$
[RIGHT][I][B]Nguồn: MathScope.ORG[/B][/I][/RIGHT]
 

thay đổi nội dung bởi: Short_list, 03-10-2014 lúc 05:53 PM
Short_list is offline   Trả Lời Với Trích Dẫn
The Following 2 Users Say Thank You to Short_list For This Useful Post:
giabao185 (10-10-2014), thaygiaocht (02-10-2014)
Old 02-10-2014, 09:00 PM   #11
Short_list
+Thành Viên+
 
Tham gia ngày: May 2012
Đến từ: Tp.HCM
Bài gởi: 85
Thanks: 12
Thanked 79 Times in 32 Posts
Trích:
Nguyên văn bởi namdung View Post
3. (Quảng Trị) Chứng minh bất đẳng thức sau $3(x^2-x+1)(y^2-y+1) \ge 2(x^2y^2-xy+1)$ với mọi x, y thực.
Bài toán là trường hợp riêng của bất đẳng thức sau đây (của Titu Andreescu)
\[3(a^2-ab+b^2)(c^2-cd+d^2) \ge 2(c^2a^2-abcd+b^2d^2).\]
[RIGHT][I][B]Nguồn: MathScope.ORG[/B][/I][/RIGHT]
 
__________________
The Simplest Solution Is The Best Solution
Short_list is offline   Trả Lời Với Trích Dẫn
The Following 4 Users Say Thank You to Short_list For This Useful Post:
huy230499 (17-01-2015), Juliel (02-10-2014), namdung (04-10-2014), thaygiaocht (02-10-2014)
Old 03-10-2014, 05:32 AM   #12
namdung
Administrator

 
Tham gia ngày: Feb 2009
Đến từ: Tp Hồ Chí Minh
Bài gởi: 1,343
Thanks: 209
Thanked 4,066 Times in 778 Posts
Gửi tin nhắn qua Yahoo chát tới namdung
Xem ra bất đẳng thức vẫn là đề tài được quan tâm nhất. Mọi người để ý giùm các bài tổ hợp, số học. Nhờ bạn nào chọn hộ các bài hình hay nhất để anh em giải và bình luận. Cuối tuần này tôi sẽ bình luận điểm qua các bài tổ hợp năm nay (rất tiến bộ là phần tổ hợp năm nay đã xuất hiện nhiều hơn, và cũng đa dạng hơn).
[RIGHT][I][B]Nguồn: MathScope.ORG[/B][/I][/RIGHT]
 
namdung is offline   Trả Lời Với Trích Dẫn
The Following User Says Thank You to namdung For This Useful Post:
thaygiaocht (04-10-2014)
Old 03-10-2014, 07:22 PM   #13
294753618
+Thành Viên+
 
Tham gia ngày: Apr 2013
Bài gởi: 2
Thanks: 0
Thanked 1 Time in 1 Post
Bài 10:
Cho $y=0\Rightarrow f(f(x)) + f(x) = 2x$
Xét dãy $ \begin{cases}u_0=x\\u_(n+1)=f(u_n),n>0\end{cases}$
suy ra $u_(n+2)=-u_(n+1)+2u_n$
Pt đặc trưng: $\gamma ^2 +\gamma-2=0$ $\Leftrightarrow \gamma=1 \vee \gamma=-2$
$\Rightarrow u_n=c_1+c_2(-2)^n$
Ta có: $\begin{cases}x=u_0=c_1+c_2 \\ f(x)=u_1=c_1-2c_2\end{cases}$
Từ đó ta được $f(x)=x$ hoặc $f(x)=c-2x$
Ta thấy không tồn tại $a$ và $b$ thỏa mãn $f(a)=a$ và $f(b)=c-2b$
Vậy có hai hàm số tm là $f(x)=x$ hoặc $f(x)=c-2x$
[RIGHT][I][B]Nguồn: MathScope.ORG[/B][/I][/RIGHT]
 

thay đổi nội dung bởi: novae, 23-10-2014 lúc 10:59 PM
294753618 is offline   Trả Lời Với Trích Dẫn
Old 03-10-2014, 10:27 PM   #14
thaygiaocht
+Thành Viên+
 
thaygiaocht's Avatar
 
Tham gia ngày: Aug 2012
Đến từ: Chuyên Hà Tĩnh
Bài gởi: 165
Thanks: 793
Thanked 216 Times in 93 Posts
Trích:
Nguyên văn bởi namdung View Post
5. (PTNK) Cho a, b, c là các số thực dương thỏa mãn điều kiện $ (a+1)(b+1)(c+1) = 1 + 4abc $ Chứng minh rằng ta có bất đẳng thức $ a + b + c \le 1 + abc $
Ta có thể tiếp cận vấn đề như sau:
Hướng 1. Bài toán quy về: Cho $ x,y,z>0 $ mà $ xy+yz+zx+x+y+z=3. $ Chứng minh $ xy+yz+zx-xyz \le 1. $
Giả sử $ z $ min thì $ 1\ge z+z^2. $ Đặt $ S=x+y;P=xy. $ Khi đó giải điều kiện $ S^2 \ge 4P $ được $ S \ge 2\dfrac{3-z}{\sqrt{z^2+z+4}+z+1}. $ Thay vào đpcm ta cần $ S \ge \dfrac{2-4z+z^2}{1-z-z^2}. $ Tóm lại ta chỉ cần chứng minh
$ 2\dfrac{3-z}{\sqrt{z^2+z+4}+z+1} \ge \dfrac{2-4z+z^2}{1-z-z^2}. $
Điều này đúng do $ \sqrt{z^2+z+4} \le \dfrac{z^2+z+8}{4} $ và $ z \in [0;1). $ Bài toán kết thúc.
Hướng 2. Bài toán quy về: Cho $ x,y,z>-1 $ thỏa $ 3xyz+2(xy+yz+zx)=3. $ Chứng minh $ xy+yz+zx \ge 0. $
+ Nếu trong 3 số $ x,y,z $ có 3 số dương thì $ xy+yz+zx \ge 0, $ đpcm.
+ Nếu trong 3 số $ x,y,z $ có 2 số dương thì $ xyz \le 0 <1, $ đpcm.
+ Nếu trong 3 số $ x,y,z $ có 0 số dương thì $ xyz \le 0 <1, $ đpcm.
+ Nếu trong 3 số $ x,y,z $ có 1 số dương. Giả sử $ x>0. $ Đổi dấu $ y, z $ ta có $ y,z \in (0;1) $ thỏa mãn $ 3xyz+2yz-2xy-2zx=3. $ Cần chứng minh $ xyz \le 1. $ Điều này hiển nhiên đúng vì $ 3>2yz $ nên $ 3yz>2y+2z. $ Rút $ x $ từ giả thiết thay vào được điều tương đương là $ (yz)^2+y+z \ge 0. $ Đây là điều hiển nhiên đúng.
Đẳng thức không xảy ra.
[RIGHT][I][B]Nguồn: MathScope.ORG[/B][/I][/RIGHT]
 
__________________
https://www.facebook.com/thaygiaocht
thaygiaocht is offline   Trả Lời Với Trích Dẫn
The Following 4 Users Say Thank You to thaygiaocht For This Useful Post:
beppkid (25-01-2015), luuvanthai (03-10-2014), namdung (04-10-2014), Raul Chavez (04-10-2014)
Old 04-10-2014, 08:03 AM   #15
thaygiaocht
+Thành Viên+
 
thaygiaocht's Avatar
 
Tham gia ngày: Aug 2012
Đến từ: Chuyên Hà Tĩnh
Bài gởi: 165
Thanks: 793
Thanked 216 Times in 93 Posts
Trích:
Nguyên văn bởi namdung View Post
Xem ra bất đẳng thức vẫn là đề tài được quan tâm nhất. Mọi người để ý giùm các bài tổ hợp, số học. Nhờ bạn nào chọn hộ các bài hình hay nhất để anh em giải và bình luận. Cuối tuần này tôi sẽ bình luận điểm qua các bài tổ hợp năm nay (rất tiến bộ là phần tổ hợp năm nay đã xuất hiện nhiều hơn, và cũng đa dạng hơn).
Tuyển tập các bài toán Hình học VMO 2015
[Only registered and activated users can see links. ]
[RIGHT][I][B]Nguồn: MathScope.ORG[/B][/I][/RIGHT]
 
File Kèm Theo
Kiểu File : doc VMO 2015 Hinh hoc.doc (2.32 MB, 793 lần tải)
__________________
https://www.facebook.com/thaygiaocht

thay đổi nội dung bởi: thaygiaocht, 04-10-2014 lúc 08:05 AM
thaygiaocht is offline   Trả Lời Với Trích Dẫn
The Following 7 Users Say Thank You to thaygiaocht For This Useful Post:
HoangHungChels (04-10-2014), huynhcongbang (07-10-2014), Juliel (04-10-2014), Livetolove2207 (10-12-2014), namdung (04-10-2014), vulalach (05-10-2014), whatever2507 (05-10-2014)
Trả lời Gởi Ðề Tài Mới

Bookmarks


Quuyền Hạn Của Bạn
You may not post new threads
You may not post replies
You may not post attachments
You may not edit your posts

BB code is Mở
Smilies đang Mở
[IMG] đang Mở
HTML đang Tắt

Chuyển đến


Múi giờ GMT. Hiện tại là 03:52 AM.


Powered by: vBulletin Copyright ©2000-2024, Jelsoft Enterprises Ltd.
Inactive Reminders By mathscope.org
[page compression: 130.37 k/147.38 k (11.54%)]